LSAT and Law School Admissions Forum

Get expert LSAT preparation and law school admissions advice from PowerScore Test Preparation.

 Administrator
PowerScore Staff
  • PowerScore Staff
  • Posts: 8917
  • Joined: Feb 02, 2011
|
#45689
Complete Question Explanation
(The complete setup for this game can be found here: lsat/viewtopic.php?t=16967)

The correct answer choice is (B)

If M is selected then K and L (the other two students) are both out. Further, from the rules and inferences Z and F are also out (neither can be with M). And you also know that U and W cannot be together, so one of them is out. That gives you all five of the non-selected people: K, L, Z, F, and U/W. From that you can see that everyone else must be selected in order to fill the five spaces. So M, G, H, X, and U/W are selected, and answer choice (B) is the correct answer.

Get the most out of your LSAT Prep Plus subscription.

Analyze and track your performance with our Testing and Analytics Package.